You are on page 1of 67









































 
         

    
36$03/((;$062/87,216


 ! " !#$ %$&' $%#$( !#$$(! 


) " *# %&%&%%#'& $ $+ &")$%   ,$)&$&%-

Page 1 of 67

1. Solution: D
Let
G

event that a viewer watched gymnastics

B event that a viewer watched baseball


S event that a viewer watched soccer
Then we want to find
c
Pr G B S 1  Pr G B S

1  Pr G  Pr B  Pr S  Pr G B  Pr G S  Pr B S  Pr G B S
1  0.28  0.29  0.19  0.14  0.10  0.12  0.08 1  0.48 0.52

-------------------------------------------------------------------------------------------------------2.

Solution: A
Let R = event of referral to a specialist
L = event of lab work
We want to find
P[RL] = P[R] + P[L] P[RL] = P[R] + P[L] 1 + P[~(RL)]
= P[R] + P[L] 1 + P[~R~L] = 0.30 + 0.40 1 + 0.35 = 0.05 .

-------------------------------------------------------------------------------------------------------3.

Solution: D
First note
P > A B@

P > A@  P > B @  P > A B @

P > A B '@ P > A@  P > B '@  P > A B '@


Then add these two equations to get
P > A B @  P > A B '@ 2 P > A@  P > B @  P > B '@  P > A B @  P > A B '@
0.7  0.9 2 P > A@  1  P A B A B '

1.6 2 P > A@  1  P > A@


P > A@ 0.6

Page 2 of 67

4.

Solution: A
For i 1, 2, let
Ri event that a red ball is drawn form urn i
Bi

event that a blue ball is drawn from urn i .

Then if x is the number of blue balls in urn 2,


0.44 Pr[ R1  R2 * B1  B2 ] Pr[ R1  R2 ]  Pr > B1  B2 @
Pr > R1 @ Pr > R2 @  Pr > B1 @ Pr > B2 @

4 16 6 x

10 x  16 10 x  16
Therefore,
32
3x
3x  32

x  16 x  16 x  16
2.2 x  35.2 3 x  32
0.8 x 3.2
x 4
2.2

-------------------------------------------------------------------------------------------------------5.

Solution: D
Let N(C) denote the number of policyholders in classification C . Then
N(Young Female Single) = N(Young Female) N(Young Female Married)
= N(Young) N(Young Male) [N(Young Married) N(Young Married
Male)] = 3000 1320 (1400 600) = 880 .

-------------------------------------------------------------------------------------------------------6.

Solution: B
Let
H = event that a death is due to heart disease
F = event that at least one parent suffered from heart disease
Then based on the medical records,
210  102 108
P H F c
937
937
937  312 625
P F c
937
937
c
P H F 108 625 108
0.173
and P H | F c
937 937 625
P F c

Page 3 of 67

7.

Solution: D
Let
A event that a policyholder has an auto policy
H event that a policyholder has a homeowners policy
Then based on the information given,
Pr A H 0.15
Pr A H c Pr A  Pr A H 0.65  0.15 0.50

Pr Ac H Pr H  Pr A H 0.50  0.15 0.35

and the portion of policyholders that will renew at least one policy is given by
0.4 Pr A H c  0.6 Pr Ac H  0.8 Pr A H

0.4 0.5  0.6 0.35  0.8 0.15

0.53

53%

-------------------------------------------------------------------------------------------------------100292
01B-9
8.
Solution: D
Let
C = event that patient visits a chiropractor
T = event that patient visits a physical therapist
We are given that
Pr >C @ Pr >T @  0.14
Pr C  T 0.22

Pr C c  T c 0.12
Therefore,
0.88 1  Pr C c  T c

Pr >C * T @ Pr >C @  Pr >T @  Pr >C  T @

Pr >T @  0.14  Pr >T @  0.22


2 Pr >T @  0.08

or

Pr >T @

0.88  0.08

2 0.48

Page 4 of 67

9.

Solution: B
Let
M event that customer insures more than one car
S event that customer insures a sports car
Then applying DeMorgans Law, we may compute the desired
probability as follows:
c
Pr M c S c Pr M S 1  Pr M S 1  Pr M  Pr S  Pr M S

1  Pr M  Pr S  Pr S M Pr M 1  0.70  0.20  0.15 0.70 0.205

-------------------------------------------------------------------------------------------------------10.

Solution: C
Consider the following events about a randomly selected auto insurance customer:
A = customer insures more than one car
B = customer insures a sports car
We want to find the probability of the complement of A intersecting the complement of B
(exactly one car, non-sports). But P ( Ac Bc) = 1 P (A B)
And, by the Additive Law, P ( A B ) = P ( A) + P ( B ) P ( A B ).
By the Multiplicative Law, P ( A B ) = P ( B _ A ) P (A) = 0.15 * 0.64 = 0.096
It follows that P ( A B ) = 0.64 + 0.20 0.096 = 0.744 and P (Ac Bc ) = 0.744 =
0.256

-------------------------------------------------------------------------------------------------------11.

Solution: B
Let
C = Event that a policyholder buys collision coverage
D = Event that a policyholder buys disability coverage
Then we are given that P[C] = 2P[D] and P[C D] = 0.15 .
By the independence of C and D, it therefore follows that
0.15 = P[C D] = P[C] P[D] = 2P[D] P[D] = 2(P[D])2
(P[D])2 = 0.15/2 = 0.075
P[D] = 0.075 and P[C] = 2P[D] = 2 0.075
Now the independence of C and D also implies the independence of CC and DC . As a
result, we see that P[CC DC] = P[CC] P[DC] = (1 P[C]) (1 P[D])
= (1 2 0.075 ) (1 0.075 ) = 0.33 .

Page 5 of 67

12.

Solution: E
Boxed numbers in the table below were computed.
High BP Low BP Norm BP

Total

Regular heartbeat
0.09
0.20
0.56
0.85
Irregular heartbeat
0.05
0.02
0.08
0.15
Total
0.14
0.22
0.64
1.00
From the table, we can see that 20% of patients have a regular heartbeat and low blood
pressure.
-------------------------------------------------------------------------------------------------------13.

Solution: C
The Venn diagram below summarizes the unconditional probabilities described in the
problem.

In addition, we are told that


P> A B C@
1
P > A B C | A B@
3
P > A B@
It follows that
1
1
x
x  0.12 x  0.04
3
3
2
x 0.04
3
x 0.06
Now we want to find
c
P A B C
c

P A B C | Ac
c

P A
1 P> A B C@
1  P > A@

x
x  0.12

1  3 0.10  3 0.12  0.06


1  0.10  2 0.12  0.06
0.28
0.60

0.467

Page 6 of 67

14.

Solution: A
1
pk = pk 1
5
f

1=

pk
k 0

11
pk  2
55
f

p0
k 05

1
... p0
5

1 1 1
pk 3
5 5 5
p0
1
1
5

kt0

5
p0
4

p0 = 4/5 .
Therefore, P[N > 1] = 1 P[N d1] = 1 (4/5 + 4/5 1/5) = 1 24/25 = 1/25 = 0.04 .
-------------------------------------------------------------------------------------------------------15.

Solution: C
A Venn diagram for this situation looks like:

We want to find w 1  x  y  z

1
1
, xz
, yz
4
3
Adding these three equations gives
We have x  y

x  y  x  z  y  z
2 x  y  z 1
x y z

5
12
1 1 5
 
4 3 12

1
2

1 1
2 2
Alternatively the three equations can be solved to give x = 1/12, y = 1/6, z =1/4
1 1 1 1
again leading to w 1   
12 6 4 2
w 1 x  y  z 1

Page 7 of 67

16.

Solution: D
Let N1 and N 2 denote the number of claims during weeks one and two, respectively.
Then since N1 and N 2 are independent,
Pr > N1  N 2

7@

7
n 0

8
29

Pr > N1

n @ Pr > N 2

7  n@

1 1
n 1 8 n
2 2
1
0 9
2
1
1
6
2
64

n 0

-------------------------------------------------------------------------------------------------------17.

Solution: D
Let
O Event of operating room charges
E Event of emergency room charges
Then
0.85 Pr O E Pr O  Pr E  Pr O E
Pr O  Pr E  Pr O Pr E

Since

So

Independence
Pr E c 0.25 1  Pr E , it follows Pr E 0.75 .
0.85 Pr O  0.75  Pr O 0.75
Pr O 1  0.75 0.10
Pr O 0.40

-------------------------------------------------------------------------------------------------------18.

Solution: D
Let X1 and X2 denote the measurement errors of the less and more accurate instruments,
respectively. If N(P,V) denotes a normal random variable with mean P and standard
deviation V, then we are given X1 is N(0, 0.0056h), X2 is N(0, 0.0044h) and X1, X2 are
X1  X 2
0.00562 h 2  0.00442 h 2
is N (0,
) = N(0,
2
4
0.00356h) . Therefore, P[0.005h d Y d 0.005h] = P[Y d 0.005h] P[Y d 0.005h] =
P[Y d 0.005h] P[Y t 0.005h]
0.005h

= 2P[Y d 0.005h] 1 = 2P Z d
 1 = 2P[Z d 1.4] 1 = 2(0.9192) 1 = 0.84.
0.00356h

independent. It follows that Y =

Page 8 of 67

19.

Solution: B
Apply Bayes Formula. Let
A Event of an accident
B1 Event the drivers age is in the range 16-20
B2 Event the drivers age is in the range 21-30
B3 Event the drivers age is in the range 30-65
B4 Event the drivers age is in the range 66-99
Then
Pr A B1 Pr B1
Pr B1 A
Pr A B1 Pr B1  Pr A B2 Pr B2  Pr A B3 Pr B3  Pr A B4 Pr B4

0.06 0.08
0.06 0.08  0.03 0.15  0.02 0.49  0.04 0.28

0.1584

--------------------------------------------------------------------------------------------------------

20.

Solution: D
Let
S = Event of a standard policy
F = Event of a preferred policy
U = Event of an ultra-preferred policy
D = Event that a policyholder dies
Then
P > D | U @ P >U @
P >U | D @
P > D | S @ P > S @  P > D | F @ P > F @  P > D | U @ P >U @

0.001 0.10
0.01 0.50  0.005 0.40  0.001 0.10

0.0141
-------------------------------------------------------------------------------------------------------21.

Solution: B
Apply Bayes Formula:
Pr Seri. Surv.

Pr Surv. Seri. Pr >Seri.@

Pr Surv. Crit. Pr > Crit.@  Pr Surv. Seri. Pr >Seri.@  Pr Surv. Stab. Pr >Stab.@

0.9 0.3
0.6 0.1  0.9 0.3  0.99 0.6

0.29

Page 9 of 67

22.

Solution: D
Let
H Event of a heavy smoker
L Event of a light smoker

N
D

Event of a non-smoker
Event of a death within five-year period

Now we are given that Pr D L

2 Pr D N and Pr D L

1
Pr D H
2

Therefore, upon applying Bayes Formula, we find that


Pr D H Pr > H @
Pr H D
Pr D N Pr > N @  Pr D L Pr > L @  Pr D H Pr > H @
2 Pr D L 0.2
0.4
1
Pr D L 0.5  Pr D L 0.3  2 Pr D L 0.2 0.25  0.3  0.4
2

0.42

-------------------------------------------------------------------------------------------------------23.

Solution: D
Let
C = Event of a collision
T = Event of a teen driver
Y = Event of a young adult driver
M = Event of a midlife driver
S = Event of a senior driver
Then using Bayes Theorem, we see that
P[C Y ]P[Y ]
P[Y~C] =
P[C T ]P[T ]  P[C Y ]P[Y ]  P[C M ]P[ M ]  P[C S ]P[ S ]
=

(0.08)(0.16)
= 0.22 .
(0.15)(0.08)  (0.08)(0.16)  (0.04)(0.45)  (0.05)(0.31)

-------------------------------------------------------------------------------------------------------24.

Solution: B
Observe

Pr N t 1 N d 4

Pr >1 d N d 4@
Pr > N d 4@

1 1 1 1 1
1
1 1 1
6  12  20  30 2  6  12  20  30

10  5  3  2
30  10  5  3  2

Page 10 of 67

20
50

2
5

25.

Solution: B
Let
Y = positive test result
D = disease is present (and ~D = not D)
Using Bayes theorem:
P[Y | D]P[ D]
(0.95)(0.01)
= 0.657 .
P[D|Y] =
P[Y | D]P[ D]  P[Y |~ D]P[~ D] (0.95)(0.01)  (0.005)(0.99)

-------------------------------------------------------------------------------------------------------26.

Solution: C
Let:
S = Event of a smoker
C = Event of a circulation problem
Then we are given that P[C] = 0.25 and P[S~C] = 2 P[S~CC]
Now applying Bayes Theorem, we find that P[C~S] =

2 P[ S C C ]P[C ]
2 P[ S C ]P[C ]  P[ S C ](1  P[C ])
C

2(0.25)
2(0.25)  0.75

P[ S C ]P[C ]
P[ S C ]P[C ]  P[ S C C ]( P[C C ])

2
23

2
.
5

-------------------------------------------------------------------------------------------------------27.

Solution: D
Use Bayes Theorem with A = the event of an accident in one of the years 1997, 1998 or
1999.
P[ A 1997]P[1997]
P[1997|A] =
P[ A 1997][ P[1997]  P[ A 1998]P[1998]  P[ A 1999]P[1999]
=

(0.05)(0.16)
= 0.45 .
(0.05)(0.16)  (0.02)(0.18)  (0.03)(0.20)

--------------------------------------------------------------------------------------------------------

Page 11 of 67

28.

Solution: A
Let
C = Event that shipment came from Company X
I1 = Event that one of the vaccine vials tested is ineffective
P > I1 | C @ P > C @
Then by Bayes Formula, P > C | I1 @
P > I1 | C @ P > C @  P I1 | C c P C c
Now
1
P >C @
5
1 4
P C c 1  P > C @ 1 
5 5
P > I1 | C @
P I1 | C c

0.10 0.90
0.02 0.98
29

30
1

30
1

Therefore,

0.141
29

0.334

0.141 1/ 5
0.141 1/ 5  0.334 4 / 5

P > C | I1 @

0.096

-------------------------------------------------------------------------------------------------------29.

Solution: C
Let T denote the number of days that elapse before a high-risk driver is involved in an
accident. Then T is exponentially distributed with unknown parameter O . Now we are
given that
50

0.3 = P[T d 50] =

Oe

 Ot

e  O t

dt

50
0

= 1 e50O

Therefore, e50O = 0.7 or O =  (1/50) ln(0.7)


80

It follows that P[T d 80] =

Oe

 Ot

dt

e  O t

80
0

= 1 e80O

(80/50) ln(0.7)

=1e

= 1 (0.7)80/50 = 0.435 .

-------------------------------------------------------------------------------------------------------30.

Solution: D
e O O 2
Let N be the number of claims filed. We are given P[N = 2] =
2!
2
4
= 4]24 O = 6 O
O2 = 4 O = 2
Therefore, Var[N] = O = 2 .

Page 12 of 67

e O O 4
3
= 3 P[N
4!

31.

Solution: D
Let X denote the number of employees that achieve the high performance level. Then X
follows a binomial distribution with parameters n 20 and p 0.02 . Now we want to
determine x such that
Pr > X ! x @ d 0.01
or, equivalently,
0.99 d Pr > X d x @

0.02 0.98
x

k 0

20
k

20  k

The following table summarizes the selection process for x:


Pr > X x @
Pr > X d x @
x
0
1
2

0.98 0.668
19
20 0.02 0.98
2
18
190 0.02 0.98
20

0.668
0.272
0.053

0.940
0.993

Consequently, there is less than a 1% chance that more than two employees will achieve
the high performance level. We conclude that we should choose the payment amount C
such that
2C 120, 000
or
C 60, 000
-------------------------------------------------------------------------------------------------------32.

Solution: D
Let
X = number of low-risk drivers insured
Y = number of moderate-risk drivers insured
Z = number of high-risk drivers insured
f(x, y, z) = probability function of X, Y, and Z
Then f is a trinomial probability function, so
Pr > z t x  2@ f 0, 0, 4  f 1, 0,3  f 0,1,3  f 0, 2, 2

0.20

 4 0.50 0.20  4 0.30 0.20 


3

0.0488

Page 13 of 67

4!
2
2
0.30 0.20
2!2!

33.

Solution: B
Note that
Pr > X ! x @

20
x

0.005 20  t dt

0.005 400  200  20 x  x 2


2

where 0  x  20 . Therefore,

Pr X ! 16 X ! 8

Pr > X ! 16@
Pr > X ! 8@

0.005 20t  t 2
2

20
x

0.005 200  20 x  x 2
2

200  20 16  1 16
2
2
1
200  20 8  8
2

8
72

1
9

-------------------------------------------------------------------------------------------------------34.

Solution: C
2
We know the density has the form C 10  x for 0  x  40 (equals zero otherwise).

First, determine the proportionality constant C from the condition

40
0

f ( x)dx 1 :

40

2
C C

C
0
10 50 25
so C 25 2 , or 12.5 . Then, calculate the probability over the interval (0, 6):
6
2
1 6
1 1
12.5 10  x dx  10  x
 12.5 0.47 .
0
0
10 16
1

40

C 10  x dx  C (10  x) 1
2

-------------------------------------------------------------------------------------------------------35.

Solution: C
Let the random variable T be the future lifetime of a 30-year-old. We know that the
density of T has the form f (x) = C(10 + x)2 for 0 < x < 40 (and it is equal to zero
otherwise). First, determine the proportionality constant C from the condition
040 f ( x)dx 1:
40
2
C
1 = f ( x)dx  C (10  x) 1 |040
0
25
25
so that C =
= 12.5. Then, calculate P(T < 5) by integrating f (x) = 12.5 (10 + x)2
2
over the interval (0.5).

Page 14 of 67


36.

Solution:B
Todeterminek,notethat
1
k
4
5
1= k 1  y dy  1  y
5
0

k

5

1
0

k=5
WenextneedtofindP[V>10,000]=P[100,000Y>10,000]=P[Y>0.1]
1

= 5 1  y dy
4

 1  y

5 1
0.1

=(0.9)5=0.59andP[V>40,000]

0.1

=P[100,000Y>40,000]=P[Y>0.4]= 5 1  y dy
4

 1  y

5 1
0.4

=(0.6)5=0.078.

0.4

ItnowfollowsthatP[V>40,000~V>10,000]
P[V ! 40, 000 V ! 10, 000] P[V ! 40, 000]
=
P[V ! 10, 000]
P[V ! 10, 000]

0.078
=0.132.
0.590




37.
Solution:D
LetTdenoteprinterlifetime.Thenf(t)=et/2,0dtd
Notethat
1
1
P[Td1]= e  t / 2 dt e  t / 2 1 =1e1/2=0.393
0
2
0
2

1
2
P[1dTd2]= e  t / 2 dt e  t / 2 1 =e1/2e1=0.239
2
1
Next,denoterefundsforthe100printerssoldbyindependentandidentically
distributedrandomvariablesY1,...,Y100where
with probability 0.393
200

Yi 100
with probability 0.239
i = 1, . . . , 100 
0
with probability 0.368

NowE[Yi]=200(0.393)+100(0.239)=102.56
100

Therefore,ExpectedRefunds= E >Yi @ =100(102.56)=10,256.


i 1




Page 15 of 67

38.

Solution: A
Let F denote the distribution function of f. Then
F x

Pr > X d x @

3t 4 dt

t 3

x
1

1  x 3

Using this result, we see


Pr > X / 2: X t 1.5@

Pr X / 2 X t 1.5
Pr > X t 1.5@

F 2  F 1.5
1  F 1.5

Pr > X / 2@  Pr > X d 1.5@


Pr > X t 1.5@

1.5  2
3
1.5
3

3

3
1
4

0.578

-------------------------------------------------------------------------------------------------------39.

Solution: E
Let X be the number of hurricanes over the 20-year period. The conditions of the
problem give x is a binomial distribution with n = 20 and p = 0.05 . It follows that
P[X < 2] = (0.95)20(0.05)0 + 20(0.95)19(0.05) + 190(0.95)18(0.05)2
= 0.358 + 0.377 + 0.189 = 0.925 .

-------------------------------------------------------------------------------------------------------40.

Solution: B
Denote the insurance payment by the random variable Y. Then
if 0  X d C
0
Y
X  C if C  X  1
Now we are given that
0.64 Pr Y  0.5

Pr 0  X  0.5  C

Therefore, solving for C, we find C r0.8  0.5


Finally, since 0  C  1 , we conclude that C 0.3

Page 16 of 67

0.5 C
0

2 x dx

0.5  C
0

0.5  C

41.

Solution: E
Let
X = number of group 1 participants that complete the study.
Y = number of group 2 participants that complete the study.
Now we are given that X and Y are independent.
Therefore,
P X t 9 Y / 9 X / 9 Y t 9

P X t 9 Y / 9  P X / 9 Y t 9

2 P X t 9 Y / 9

(due to symmetry)

2 P > X t 9 @ P >Y / 9 @

2 P > X t 9@ P > X / 9@

(again due to symmetry)

2 P > X t 9@ 1  P > X t 9@
9
10
9
10
2 10
0.2 0.8  10 0.8 1  10 0.2 0.8  10 0.8
9 10 9 10
2 > 0.376@>1  0.376@ 0.469

-------------------------------------------------------------------------------------------------------42.

Solution: D
Let
IA = Event that Company A makes a claim
IB = Event that Company B makes a claim
XA = Expense paid to Company A if claims are made
XB = Expense paid to Company B if claims are made
Then we want to find
Pr I AC I B I A I B X A / X B

Pr I AC I B  Pr I A I B X A / X B

Pr I AC Pr > I B @  Pr > I A @ Pr > I B @ Pr > X A / X B @

(independence)

0.60 0.30  0.40 0.30 Pr > X B  X A t 0@


0.18  0.12 Pr > X B  X A t 0@
Now X B  X A is a linear combination of independent normal random variables.
Therefore, X B  X A is also a normal random variable with mean
M E > X B  X A @ E > X B @  E > X A @ 9, 000  10, 000 1, 000
and standard deviation V

Var X B  Var X A

It follows that

Page 17 of 67

2000  2000
2

2000 2

1000

Pr > X B  X A t 0@ Pr Z t
2000 2

Pr Z t
2 2

(Z is standard normal)

1  Pr Z /
2 2

1  Pr > Z  0.354@
1  0.638 0.362

Finally,

Pr I AC I B I A I B X A / X B

0.18  0.12 0.362


0.223

-------------------------------------------------------------------------------------------------------43.

Solution: D
If a month with one or more accidents is regarded as success and k = the number of
failures before the fourth success, then k follows a negative binomial distribution and the
requested probability is
4
k
3
3 k 3 2
Pr > k t 4@ 1  Pr > k d 3@ 1  k
5 5
k 0
3
1
5

3 2 0 4 2 1 5 2 2 6 2 3
0  1  2  3
5
5
5
5

3 8 8 32
1  1   
5 5 5 25
0.2898
Alternatively the solution is
4

2
4 2 3
5 2 3
6 2 3
 1  2  3 0.2898
5
5 5
5 5
5 5
which can be derived directly or by regarding the problem as a negative binomial
distribution with
i) success taken as a month with no accidents
ii) k = the number of failures before the fourth success, and
iii) calculating Pr > k d 3@

Page 18 of 67

44.

Solution: C
If k is the number of days of hospitalization, then the insurance payment g(k) is
100k
for k 1, 2, 3
g(k) =
300  50 (k  3) for k 4, 5.

^
5

Thus, the expected payment is

g (k ) p

100 p1  200 p2  300 p3  350 p4  400 p5 =

k 1

1
100 u 5  200 u 4  300 u 3  350 u 2  400 u 1 =220
15
-------------------------------------------------------------------------------------------------------45.

Solution: D
0

2
4 x
x2
x3
x3




dx
dx
2 10 0 10
30 2 30 0

Note that E X

8 64

30 30

56
30

28
15

-------------------------------------------------------------------------------------------------------46.

Solution: D
The density function of T is
1 t / 3
f t
e
, 0/ t / f
3
Therefore,
E > X @ E max T , 2

f t
2 t / 3
e dt  e t / 3 dt
2
3
3
f

2e  t / 3 : 02 te  t / 3 : f2  e t / 3 dt
2

2e

2 / 3

 2  2e

2 / 3

 3e t / 3 : f2

2  3e 2 / 3

Page 19 of 67

47.

Solution: D
Let T be the time from purchase until failure of the equipment. We are given that T is
exponentially distributed with parameter O = 10 since 10 = E[T] = O . Next define the
payment
for 0 d T d 1
x
x

for 1  T d 3
P under the insurance contract by P
2

for T ! 3
0
We want to find x such that
1
x t/10
e
dt +
1000 = E[P] =
10
0

x 1 t/10
 t /10
1 2 10 e dt =  xe

= x e
+ x (x/2) e
+ (x/2) e
We conclude that x = 5644 .
1/10

3/10

1/10

1/10

= x(1 e

x
 e t /10
2

3
1

e3/10) = 0.1772x .

-------------------------------------------------------------------------------------------------------48.

Solution: E
Let X and Y denote the year the device fails and the benefit amount, respectively. Then
the density function of X is given by
x 1
f x 0.6 0.4 , x 1, 2,3...
and
1000 5  x if x 1, 2,3, 4

if x ! 4
0
It follows that
2
3
E >Y @ 4000 0.4  3000 0.6 0.4  2000 0.6 0.4  1000 0.6 0.4
y

2694
-------------------------------------------------------------------------------------------------------49.

Solution: D
Define f ( X ) to be hospitalization payments made by the insurance policy. Then
f (X )

100 X

300  25 X  3

if X

1, 2,3

if X

4,5

and

Page 20 of 67

f k Pr > X

E f X

k 1

k@

5
4
3
2
1
100  200  300  325  350
15
15
15
15
15
1
640
213.33
>100  160  180  130  70@
3
3

-------------------------------------------------------------------------------------------------------50.

Solution: C
Let N be the number of major snowstorms per year, and let P be the amount paid to
(3 / 2) n e 3/ 2
, n = 0, 1, 2, . . . and
the company under the policy. Then Pr[N = n] =
n!
for N 0
0
P
.
10, 000( N  1) for N t 1
f

Now observe that E[P] =

10, 000(n  1)
n 1

(3 / 2) n e 3/ 2
n!

(3 / 2) n e 3/ 2
= 10,000 e3/2 + E[10,000 (N 1)]
n!
n 0
3/2
= 10,000 e + E[10,000N] E[10,000] = 10,000 e3/2 + 10,000 (3/2) 10,000 = 7,231 .

= 10,000 e3/2 +

10, 000(n  1)

-------------------------------------------------------------------------------------------------------51.

Solution: C
Let Y denote the manufacturers retained annual losses.
for 0.6  x d 2
x
Then Y
for x ! 2
2
f
2.5(0.6) 2.5
2.5(0.6) 2.5

x
dx
2
x3.5 2 x3.5 dx
0.6
2

and E[Y] =
=

2.5(0.6) 2.5
1.5 x1.5

2
0.6

2(0.6) 2.5
(2) 2.5

2.5(0.6) 2.5
2(0.6) 2.5

dx
x 2.5
x 2.5
0.6

2.5(0.6) 2.5 2.5(0.6) 2.5 (0.6) 2.5



 1.5 = 0.9343 .
1.5(2)1.5
1.5(0.6)1.5
2

Page 21 of 67

f
2

52.

Solution: A
Let us first determine K. Observe that
1 1 1 1
60  30  20  15  12
137
1 K 1     K
K

60
2 3 4 5

60
60
K
137
It then follows that
Pr > N n @ Pr N n Insured Suffers a Loss Pr > Insured Suffers a Loss @
60
3
, N 1,...,5
0.05
137 N
137 N
Now because of the deductible of 2, the net annual premium P
X

N  2

E>X @

E > X @ where

, if N d 2
, if N ! 2

Then,

3
3
3
1
N  2 137 N 1 137  2 137 4  3 137 5
5

N 3

0.0314

-------------------------------------------------------------------------------------------------------53.

Solution: D
Let W denote claim payments. Then W
10

for 1  y d 10
for y t 10

10

2
2
It follows that E[W] = y 3 dy  10 3 dy
y
y
1
10

Page 22 of 67

2 10
10 f
 2
y1
y 10

= 2 2/10 + 1/10 = 1.9 .

54.

Solution: B
Let Y denote the claim payment made by the insurance company.
Then
with probability 0.94
0

Y Max 0, x  1 with probability 0.04


14
with probability 0.02

and
E >Y @

0.94 0  0.04 0.5003 1 x  1 e x / 2 dx  0.02 14


15

0.020012 1

xe  x / 2 dx  e  x / 2 dx  0.28

1
15
15
0.28  0.020012 2 xe  x / 2 : 15
2 e  x / 2 dx  e  x / 2 dx
1
1
1

15

15

15
0.28  0.020012 30e 7.5  2e 0.5  e x / 2 dx

0.28  0.020012 30e 7.5  2e 0.5  2e x / 2 : 15


1

0.28  0.020012 30e 7.5  2e 0.5  2e7.5  2e0.5


0.28  0.020012 32e 7.5  4e 0.5

0.28  0.020012 2.408

0.328
(in thousands)
It follows that the expected claim payment is 328 .
-------------------------------------------------------------------------------------------------------55.

Solution: C
k
, 0 < x < f . To find k, note
(1  x) 4

The pdf of x is given by f(x) =


f

k
0 (1  x)4 dx
k=3
1=

k 1
3 (1  x)3
f

It then follows that E[x] =

f
0

k
3

3x

(1  x)

dx and substituting u = 1 + x, du = dx, we see

3(u  1)
E[x] =
du = 3 (u 3  u 4 )du
4
u
1
1

u 2 u 3
3


2 3 1

Page 23 of 67

1 1
3  = 3/2 1 = .
2 3

56.

Solution: C
Let Y represent the payment made to the policyholder for a loss subject to a deductible D.
for 0 d X d D
0
That is Y
x  D for D  X d 1
Then since E[X] = 500, we want to choose D so that
1000
1
1
1 ( x  D) 2 1000 (1000  D) 2
500
( x  D)dx
D
4
1000
1000
2
2000
D
(1000 D)2 = 2000/4 500 = 5002
1000 D = r 500
D = 500 (or D = 1500 which is extraneous).

-------------------------------------------------------------------------------------------------------57.

Solution: B
1
for the claim size X in a certain class of accidents.
(1  2500t ) 4
(4)(2500)
10, 000
First, compute Mxc(t) =
5
(1  2500t )
(1  2500t )5
(10, 000)(5)(2500) 125, 000, 000
Mxs(t) =
(1  2500t )6
(1  2500t )6
Then E[X] = Mxc (0) = 10,000
E[X2] = Mxs (0) = 125,000,000
Var[X] = E[X2] {E[X]}2 = 125,000,000 (10,000)2 = 25,000,000
Var[ X ] = 5,000 .

We are given that Mx(t) =

-------------------------------------------------------------------------------------------------------58.

Solution: E
Let XJ, XK, and XL represent annual losses for cities J, K, and L, respectively. Then
X = XJ + XK + XL and due to independence
x x x t
M(t) = E e xt E e J K L E e xJ t E e xK t E e xLt
= MJ(t) MK(t) ML(t) = (1 2t)3 (1 2t)2.5 (1 2t)4.5 = (1 2t)10
Therefore,
Mc(t) = 20(1 2t)11
Ms(t) = 440(1 2t)12
Msc(t) = 10,560(1 2t)13
E[X3] = Msc(0) = 10,560

Page 24 of 67

59.

Solution: B
The distribution function of X is given by
F x

2.5 200
200 t 3.5

2.5

dt

 200
t 2.5

2.5 x

200
1

200

x 2.5

2.5

x ! 200

th

Therefore, the p percentile x p of X is given by

p
100

F xp

xp

200

1  0.01 p

xp

1  0.01 p
xp

200
1

25

2.5

2.5

2.5

2.5

200
xp

200

1  0.01 p

It follows that x 70  x 30

25

200

0.30

25

200

0.70

25

93.06

-------------------------------------------------------------------------------------------------------60.

Solution: E
Let X and Y denote the annual cost of maintaining and repairing a car before and after
the 20% tax, respectively. Then Y = 1.2X and Var[Y] = Var[1.2X] = (1.2)2 Var[X] =
(1.2)2(260) = 374 .

-------------------------------------------------------------------------------------------------------61.

Solution: A
The first quartile, Q1, is found by =

Q1

f(x) dx . That is, = (200/Q1)2.5 or

Q1 = 200 (4/3)0.4 = 224.4 . Similarly, the third quartile, Q3, is given by Q3 = 200 (4)0.4
= 348.2 . The interquartile range is the difference Q3 Q1 .

Page 25 of 67

62.

Solution: C
First note that the density function of X is given by
1
if
x 1
2

1 x  2
f x x  1 if

otherwise
0
Then

E X

2
1
 x x  1 dx
2 1

1 8 4 1 1
   
2 3 2 3 2

E X

2
1
 x 2 x  1 dx
2 1

 E X

E X

1 1 3 1 2
 x  x
2 3
2 1

7
4
1
3
3

2
1
 x3  x 2 dx
2 1

1 16 8 1 1
   
2 4 3 4 3

Var X

2
1
 x 2  x dx
2 1

17 7

4 3
2

1 1 4 1 3
 x  x
2 4
3 1

23
12

23 4

12 3

23 16

12 9

5
36

-------------------------------------------------------------------------------------------------------63.

Solution: C
X if 0 d X d 4
Note Y
4 if 4 / X d 5
Therefore,
41
54
1 2 4 4 5
E >Y @ xdx  dx
x : 0  x: 4
0 5
4 5
10
5
16 20 16 8 4 12
 

10 5 5 5 5 5
41
5 16
1 3 4 16 5
E Y 2 x 2 dx 
dx
x : 0  x: 4
0 5
4 5
15
5
64 80 64 64 16 64 48 112
 


15 5 5 15 5 15 15 15

Var >Y @

E Y 2  E >Y @

112 12

15 5

1.71

Page 26 of 67

64.

Solution: A
Let X denote claim size. Then E[X] = [20(0.15) + 30(0.10) + 40(0.05) + 50(0.20) +
60(0.10) + 70(0.10) + 80(0.30)] = (3 + 3 + 2 + 10 + 6 + 7 + 24) = 55
E[X2] = 400(0.15) + 900(0.10) + 1600(0.05) + 2500(0.20) + 3600(0.10) + 4900(0.10)
+ 6400(0.30) = 60 + 90 + 80 + 500 + 360 + 490 + 1920 = 3500
Var[X] = E[X2] (E[X])2 = 3500 3025 = 475 and Var[ X ] = 21.79 .
Now the range of claims within one standard deviation of the mean is given by
[55.00 21.79, 55.00 + 21.79] = [33.21, 76.79]
Therefore, the proportion of claims within one standard deviation is
0.05 + 0.20 + 0.10 + 0.10 = 0.45 .

-------------------------------------------------------------------------------------------------------65.

Solution: B
Let X and Y denote repair cost and insurance payment, respectively, in the event the auto
is damaged. Then
if x d 250
0
Y
x  250 if x ! 250
and
2
1500
1
1
2 1500 1250
E >Y @
x

250
dx
x

250
521



250
250 1500
3000
3000
1500
1
1
12503
2
3
E Y 2
x  250 dx
x  250 1500
434, 028


250
250 1500
4500
4500
E Y 2  ^ E >Y @`

Var >Y @
Var >Y @

434, 028  521

403

-------------------------------------------------------------------------------------------------------66.

Solution: E
Let X1, X2, X3, and X4 denote the four independent bids with common distribution
function F. Then if we define Y = max (X1, X2, X3, X4), the distribution function G of Y is
given by
G y Pr >Y d y @

Pr X 1 d y X 2 d y X 3 d y X 4 d y
Pr > X 1 d y @ Pr > X 2 d y @ Pr > X 3 d y @ Pr > X 4 d y @
F y
1
3
5
4
d yd
1  sinS y ,
16
2
2
It then follows that the density function g of Y is given by
4

Page 27 of 67

g y G ' y
1
3
1  sinS y S cosS y
4

cosS y 1  sinS y

3
5
d yd
2
2

Finally,
E >Y @

yg y dy

5/ 2

5/ 2

3/ 2

3/ 2

ycosS y 1  sinS y dy
3

-------------------------------------------------------------------------------------------------------67.

Solution: B
The amount of money the insurance company will have to pay is defined by the random
variable
1000 x if x  2
Y
if x t 2
2000
where x is a Poisson random variable with mean 0.6 . The probability function for X is
k
e 0.6 0.6
p x
k 0,1, 2,3" and
k!
k
f 0.6
0.6
0.6
E >Y @ 0  1000 0.6 e  2000e k 2
k!
k

f 0.6
1000 0.6 e 0.6  2000 e 0.6 k 0
 e0.6  0.6 e0.6
k!

2000e

0.6

0.6
k!

k 0

 2000e 0.6  1000 0.6 e 0.6

2000  2000e 0.6  600e0.6

573
E Y 2

1000 0.6 e0.6  2000


2

e0.6 k

0.6k
2
k!

0.6k
2
2
2
 2000 e 0.6  2000  1000 0.6 e 0.6
0

k!
2
2
2
 2000 e 0.6  2000  1000 0.6 e 0.6

2000 e0.6 k
2

2000

816,893
Var >Y @
Var >Y @

E Y 2  ^ E >Y @`

816,893  573

699

Page 28 of 67

488,564

68.

Solution: C
Note that X has an exponential distribution. Therefore, c = 0.004 . Now let Y denote the
for x  250
x
claim benefits paid. Then Y
and we want to find m such that 0.50
for x t 250
250
m

= 0.004e 0.004 x dx

e 0.004 x

= 1 e0.004m

This condition implies e0.004m = 0.5 m = 250 ln 2 = 173.29 .


-------------------------------------------------------------------------------------------------------69.

Solution: D
The distribution function of an exponential random variable
T with parameter T is given by F t 1  e  t T , t ! 0

Since we are told that T has a median of four hours, we may determine T as follows:
1
F 4 1  e4 T
2
1
e 4 T
2
4
 ln 2 

4
ln 2

Therefore, Pr T t 5 1  F 5 e 5 T

5ln 2
4

25 4

0.42

-------------------------------------------------------------------------------------------------------70.

Solution: E
Let X denote actual losses incurred. We are given that X follows an exponential
distribution with mean 300, and we are asked to find the 95th percentile of all claims that
exceed 100 . Consequently, we want to find p95 such that
Pr[100  x  p95 ] F ( p95 )  F (100)
0.95 =
where F(x) is the distribution function of X .
P[ X ! 100]
1  F (100)
Now F(x) = 1 ex/300 .
1  e  p95 / 300  (1  e 100 / 300 ) e 1/ 3  e  p95 / 300
1  e1/ 3e  p95 / 300
Therefore, 0.95 =
1  (1  e 100 / 300 )
e 1/ 3
e  p95 / 300 = 0.05 e 1/3
p95 = 300 ln(0.05 e1/3) = 999

Page 29 of 67

71.

Solution: A
The distribution function of Y is given by
G y Pr T 2 d y Pr T d y
F y

1 4 y

for y ! 4 . Differentiate to obtain the density function g y


Alternate solution:
Differentiate F t to obtain f t 8t 3 and set y
g y

f t y dt dy

y dtd y

4 y 2

t 2 . Then t

8 y 3 2 y 1 2
2

y and

4 y 2

-------------------------------------------------------------------------------------------------------72.

Solution: E
We are given that R is uniform on the interval 0.04, 0.08 and V

10, 000e R

Therefore, the distribution function of V is given by


F v Pr >V d v @ Pr 10, 000e R d v Pr R d ln v  ln 10, 000
1 ln v ln 10,000
dr
0.04 0.04

1
r
0.04

ln v  ln 10,000

25ln v  25ln 10, 000  1

0.04

 0.04
25 ln

10, 000

-------------------------------------------------------------------------------------------------------73.

Solution: E
F y

Pr >Y d y @

Pr 10 X 0.8 d y

Pr X d Y
10

Therefore, f y

Fc y

1 Y 4  Y 10 5 4
e
8 10

Page 30 of 67

10

 Y

10
1
e


10

74.

Solution: E
First note R = 10/T . Then
10

FR(r) = P[R d r] = P d r
T

10

10
P T t 1  FT . Differentiating with respect to
r

10
d
10
r fR(r) = FcR(r) = d/dr 1  FT  FT t 2
r
dt
r

d
1
since T is uniformly distributed on [8, 12] .
FT (t ) fT (t )
dt
4
1 10
5
Therefore fR(r) =
.
2
4 r 2r 2
-------------------------------------------------------------------------------------------------------75.

Solution: A
Let X and Y be the monthly profits of Company I and Company II, respectively. We are
given that the pdf of X is f . Let us also take g to be the pdf of Y and take F and G to be
the distribution functions corresponding to f and g . Then G(y) = Pr[Y d y] = P[2X d y]
= P[X d y/2] = F(y/2) and g(y) = Gc(y) = d/dy F(y/2) = Fc(y/2) = f(y/2) .

-------------------------------------------------------------------------------------------------------76.

Solution: A
First, observe that the distribution function of X is given by
x 3
1
1
F x 4 dt  3 : 1x 1  3 , x ; 1
1 t
t
x
Next, let X1, X2, and X3 denote the three claims made that have this distribution. Then if
Y denotes the largest of these three claims, it follows that the distribution function of Y is
given by
G y Pr > X 1 d y @ Pr > X 2 d y @ Pr > X 3 d y @
3

1
, y;1
1  3
y
while the density function of Y is given by
2

g y G ' y

1 3 9
1
3 1  3 4 4 1  3
y
y y y

Therefore,

Page 31 of 67

, y;1

E >Y @

9
1
1  3 dy
3
y y

9
2
1
1  3  6 dy
3
y y
y
f

9 18 9
9
18
9
1 y 3  y 6  y 9 dy  2 y 2  5 y 5  8 y8
1
1 2 1
9   2.025 (in thousands)
2 5 8
f

-------------------------------------------------------------------------------------------------------77.

Solution: D
Prob. = 1

1
( x  y )dxdy = 0.625
8

Note
Pr X d 1 * Y d 1

Pr X ! 1  Y ! 1

1  Pr X ! 1  Y ! 1

1

1 2
2
2
y  2  y  1 dy

16 1
18 30
1
0.625
48 48

1

1

2
1

(De Morgan's Law)

1
8 x  y dxdy
2

1
3
3
y  2  y  1

48

1 21
2
x  y 12 dy

8 1 2
1
1  64  27  27  8
48

1
2
1

-------------------------------------------------------------------------------------------------------78.

Solution: B
That the device fails within the first hour means the joint density function must be
integrated over the shaded region shown below.

This evaluation is more easily performed by integrating over the unshaded region and
subtracting from 1.

Page 32 of 67

Pr X  1 Y  1
3

1

2
3 x  2 xy
x y
1 3
dx dy 1 
dy 1  9  6 y  1  2 y dy
1
27
54 1
54 1
3

1

1 3
1
1
32
8  4 y dy 1  8 y  2 y 2 1  24  18  8  2 1 

54 1
54
54
54
1

11
27

0.41

-------------------------------------------------------------------------------------------------------79.

Solution: E
The domain of s and t is pictured below.

Note that the shaded region is the portion of the domain of s and t over which the device
fails sometime during the first half hour. Therefore,
1/ 2 1
1 1/ 2

1
1
Pr S d T d f s, t dsdt  f s, t dsdt
0 0
2
2 0 1/ 2

(where the first integral covers A and the second integral covers B).
-------------------------------------------------------------------------------------------------------80.

Solution: C
By the central limit theorem, the total contributions are approximately normally
distributed with mean nP 2025 3125 6,328,125 and standard deviation

V n

250 2025 11, 250 . From the tables, the 90th percentile for a standard normal
random variable is 1.282 . Letting p be the 90th percentile for total contributions,
p  nP
1.282, and so p nP  1.282 V n 6,328,125  1.282 11, 250 6,342,548 .
V n

Page 33 of 67

-------------------------------------------------------------------------------------------------------81.
Solution: C
1
Let X1, . . . , X25 denote the 25 collision claims, and let X
(X1 + . . . +X25) . We are
25
given that each Xi (i = 1, . . . , 25) follows a normal distribution with mean 19,400 and
standard deviation 5000 . As a result X also follows a normal distribution with mean
1
19,400 and standard deviation
(5000) = 1000 . We conclude that P[ X > 20,000]
25
X  19, 400 20, 000  19, 400
X  19, 400

!
! 0.6 = 1  )(0.6) = 1 0.7257
= P
P

1000
1000

1000

= 0.2743 .
-------------------------------------------------------------------------------------------------------82.

Solution: B
Let X1, . . . , X1250 be the number of claims filed by each of the 1250 policyholders.
We are given that each Xi follows a Poisson distribution with mean 2 . It follows that
E[Xi] = Var[Xi] = 2 . Now we are interested in the random variable S = X1 + . . . + X1250 .
Assuming that the random variables are independent, we may conclude that S has an
approximate normal distribution with E[S] = Var[S] = (2)(1250) = 2500 .
Therefore P[2450 < S < 2600] =
S  2500
2450  2500 S  2500 2600  2500

P


 2
P 1 
50
2500
2500
2500

S  2500

S  2500

P
 1
 2  P
50

50

S  2500
, we have P[2450 < S < 2600]
Then using the normal approximation with Z =
50
P[Z < 2] P[Z > 1] = P[Z < 2] + P[Z < 1] 1 0.9773 + 0.8413 1 = 0.8186 .

-------------------------------------------------------------------------------------------------------83.

Solution: B
Let X1,, Xn denote the life spans of the n light bulbs purchased. Since these random
variables are independent and normally distributed with mean 3 and variance 1, the
random variable S = X1 + + Xn is also normally distributed with mean
P 3n
and standard deviation
V
n
Now we want to choose the smallest value for n such that
S  3n 40  3n
0.9772 d Pr > S ; 40@ Pr
;

n
n
This implies that n should satisfy the following inequality:

Page 34 of 67

40  3n
n
To find such an n, lets solve the corresponding equation for n:
40  3n
2
n
2 t

2 n

40  3n

3n  2 n  40 0

n  10

n 4

n 4
n 16
-------------------------------------------------------------------------------------------------------84.

Solution: B
Observe that
E>X Y@

E > X @  E >Y @ 50  20 70

Var > X  Y @ Var > X @  Var >Y @  2 Cov > X , Y @ 50  30  20 100


for a randomly selected person. It then follows from the Central Limit Theorem that T is
approximately normal with mean
E >T @ 100 70 7000

and variance
Var >T @ 100 100 1002
Therefore,
T  7000 7100  7000
Pr >T  7100@ Pr


100
100
Pr > Z  1@ 0.8413
where Z is a standard normal random variable.

Page 35 of 67

-------------------------------------------------------------------------------------------------------85.
Solution: B
Denote the policy premium by P . Since x is exponential with parameter 1000, it follows
from what we are given that E[X] = 1000, Var[X] = 1,000,000, Var[ X ] = 1000 and P =
100 + E[X] = 1,100 . Now if 100 policies are sold, then Total Premium Collected =
100(1,100) = 110,000
Moreover, if we denote total claims by S, and assume the claims of each policy are
independent of the others then E[S] = 100 E[X] = (100)(1000) and Var[S] = 100 Var[X]
= (100)(1,000,000) . It follows from the Central Limit Theorem that S is approximately
normally distributed with mean 100,000 and standard deviation = 10,000 . Therefore,
110, 000  100, 000

P[S t 110,000] = 1 P[S d 110,000] = 1 P Z d


= 1 P[Z d 1] = 1
10, 000

0.841 | 0.159 .
-------------------------------------------------------------------------------------------------------86.

Solution: E
Let X 1 ,..., X 100 denote the number of pensions that will be provided to each new recruit.
Now under the assumptions given,
0 with probability 1  0.4 0.6

X i 1 with probability 0.4 0.25 0.1

2 with probability 0.4 0.75 0.3


for i 1,...,100 . Therefore,
E > Xi @
2
E X i

0 0.6  1 0.1  2 0.3 0.7 ,


2
2
2
0 0.6  1 0.1  2 0.3 1.3 ,

and

2
2
Var > X i @ E X i  ^ E > X i @` 1.3  0.7 0.81
Since X 1 ,..., X 100 are assumed by the consulting actuary to be independent, the Central
2

Limit Theorem then implies that S X 1  ...  X 100 is approximately normally distributed
with mean
E > S @ E > X 1 @  ...  E > X 100 @ 100 0.7 70
and variance
Var > S @ Var > X 1 @  ...  Var > X 100 @ 100 0.81 81
Consequently,
S  70 90.5  70
Pr > S d 90.5@ Pr
d

9
9
Pr > Z d 2.28@
0.99

Page 36 of 67

-------------------------------------------------------------------------------------------------------87.
Solution: D
Let X denote the difference between true and reported age. We are given X is uniformly
distributed on (2.5,2.5) . That is, X has pdf f(x) = 1/5, 2.5 < x < 2.5 . It follows that
P x = E[X] = 0
2.5

Vx2 = Var[X] = E[X2] =

x2
2.5 5 dx

x3
15

2(2.5)3
=2.083
15

2.5

2.5

Vx =1.443
Now X 48 , the difference between the means of the true and rounded ages, has a
1.443
=
distribution that is approximately normal with mean 0 and standard deviation
48
0.2083 . Therefore,
1
0.25
1
0.25
= P[1.2 d Z d 1.2] = P[Z d 1.2] P[Z d
P  d X 48 d P
dZd
4
0.2083
4
0.2083
1.2]
= P[Z d 1.2] 1 + P[Z d 1.2] = 2P[Z d 1.2] 1 = 2(0.8849) 1 = 0.77 .
-------------------------------------------------------------------------------------------------------88.

Solution: C
Let X denote the waiting time for a first claim from a good driver, and let Y denote the
waiting time for a first claim from a bad driver. The problem statement implies that the
respective distribution functions for X and Y are
F x 1  e x / 6 , x ; 0
and
G y 1  e y / 3 , y ! 0

Therefore,
Pr X d 3 Y d 2
F 3 G 2

Pr > X d 3@ Pr >Y d 2@

1  e 1  e
1/ 2

2 / 3

1  e2 / 3  e1/ 2  e7 / 6

Page 37 of 67

89.

Solution: B
6
(50  x  y )

125, 000
0

We are given that f ( x, y )

for 0  x  50  y  50
otherwise

and we want to determine P[X > 20 Y > 20] . In order to determine integration limits,
consider the following diagram:
y
50

x>20 y>20
(20, 30)
(30, 20)

50

30 50  x

6
We conclude that P[X > 20 Y > 20] =
125, 000 20

(50  x  y ) dy dx .

20

-------------------------------------------------------------------------------------------------------90.

Solution: C
Let T1 be the time until the next Basic Policy claim, and let T2 be the time until the next
Deluxe policy claim. Then the joint pdf of T1 and T2 is
1
1
1  t1 / 2  t2 / 3
f (t1 , t2 ) e t1 / 2 e t2 / 3
e e
, 0 < t1 < f , 0 < t2 < f and we need to find
2
3
6
f t1

P[T2 < T1] =

1 t / 2 t / 3
0 0 6e 1 e 2 dt2 dt1

1
1

= e  t1 / 2  e  t1 / 2 e t1 / 3 dt1
2
2

0
= 0.4 .

 2 e

 t1 / 2  t2 / 3

t1
dt1
0

f
1 t1 / 2 1 5t1 / 6
 t1 / 2 3 5t1 / 6
0 2 e  2 e dt1 = e  5 e 0

1

-------------------------------------------------------------------------------------------------------91.

Solution: D
We want to find P[X + Y > 1] . To this end, note that P[X + Y > 1]
2x  2  y
=
dydx
4
0 1 x
1 2

1
1 2
1
0 2 xy  2 y  8 y 1 x dx

1 1
1
1

2
0 x  1  2  2 x(1  x)  2 (1  x)  8 (1  x) dx =
1

8 x
0

x  2 x
0

3
1
3
1
5 3 1
5
x  dx = x 3  x 2  x =
 
8
8 0 24 8 8
4
8
24

Page 38 of 67

1 1
1
  x  x 2 dx
8 4
8

17
24

3
5

2
5

92.

Solution: B
Let X and Y denote the two bids. Then the graph below illustrates the region over which
X and Y differ by less than 20:

Based on the graph and the uniform distribution:


Pr X  Y  20

Shaded Region Area

2200  2000

1
2
180
2
2002

2002  2

1802
2
1  0.9 0.19
2
200
More formally (still using symmetry)
Pr X  Y  20 1  Pr X  Y t 20 1  2 Pr > X  Y t 20@
1

2200
1
1
x  20
dydx 1  2
y 2000
dx
2
2020 2000 200
2020 200 2
2200
2
1
2
1
x  20  2000 dx 1 
x  2020
2 2020
2
200
200

1  2

2200

180
1

200

x  20

0.19

Page 39 of 67

2200
2020

-------------------------------------------------------------------------------------------------------93.

Solution: C
Define X and Y to be loss amounts covered by the policies having deductibles of 1 and 2,
respectively. The shaded portion of the graph below shows the region over which the
total benefit paid to the family does not exceed 5:

We can also infer from the graph that the uniform random variables X and Y have joint
1
density function f x, y
, 0  x  10 , 0  y  10
100
We could integrate f over the shaded region in order to determine the desired probability.
However, since X and Y are uniform random variables, it is simpler to determine the
portion of the 10 x 10 square that is shaded in the graph above. That is,
Pr Total Benefit Paid Does not Exceed 5
Pr 0  X  6, 0  Y  2  Pr 0  X  1, 2  Y  7  Pr 1  X  6, 2  Y  8  X

6 2  1 5  1 2 5 5
100

100

100

12
5 12.5


0.295
100 100 100

-------------------------------------------------------------------------------------------------------94.

Solution: C
Let f t1 , t2 denote the joint density function of T1 and T2 . The domain of f is pictured
below:

Now the area of this domain is given by


1
2
A 62  6  4 36  2 34
2

Page 40 of 67

Consequently, f t1 , t2

and
E >T1  T2 @

34
0

E >T1 @  E >T2 @

, 0  t1  6 , 0  t2  6 , t1  t2  10
elsewhere

2 E >T1 @

(due to symmetry)
6
4 t

t
2 t1 2 60 dt1  t1 2
4
34
0 34

6 1
6
10 t1 1
4

2 t1
dt2 dt1  t1
dt2 dt1
0
0 34
4
0
34

3t 2
2 1
34

6 1
4 3t1

2
dt1 
10t1  t12 dt1

4 34
0 17

4
0

10 t1
0


dt1

1 2 1 3 6
5t1  t1 4
34
3

64
24 1
2  180  72  80  5.7
3
17 34

-------------------------------------------------------------------------------------------------------95.

Solution: E
M t1 , t2
E e 1

E et1W t2 Z

t  t2 X

E e 1

t X Y  t 2 Y  X

E e t1 t2 Y e 2

t1 t2 2

e2

t1  t2 2

E e 1

t  t2 X

e2

2
2
1  2 t1t2  t2

e 1

t  t2 Y

12 t
e

2
2
1  2 t1t2  t2

et1 t2

-------------------------------------------------------------------------------------------------------96.

Solution: E
Observe that the bus driver collect 21x50 = 1050 for the 21 tickets he sells. However, he
may be required to refund 100 to one passenger if all 21 ticket holders show up. Since
passengers show up or do not show up independently of one another, the probability that
21
21
all 21 passengers will show up is 1  0.02
0.98 0.65 . Therefore, the tour

operators expected revenue is 1050  100 0.65 985 .

Page 41 of 67

97.

Solution: C
We are given f(t1, t2) = 2/L2, 0 d t1 d t2 d L .
L t2
2
2 2
2
2
Therefore, E[T1 + T2 ] = (t1  t2 ) 2 dt1dt2 =
L
0 0
t2
3
L
t1
2
2
 t2 t1 dt1
2
0 3
L
0
4 L
L
2 4 3
2 t2
= 2 t2 dt2

L 03
L2 3 0
t2

2
L2

3
L
t2

3
 t2 dt2
0 3

2 2
L
3

(L, L)

t1

-------------------------------------------------------------------------------------------------------98.

Solution: A
Let g(y) be the probability function for Y = X1X2X3 . Note that Y = 1 if and only if
X1 = X2 = X3 = 1 . Otherwise, Y = 0 . Since P[Y = 1] = P[X1 = 1 X2 = 1 X3 = 1]
= P[X1 = 1] P[X2 = 1] P[X3 = 1] = (2/3)3 = 8/27 .
19
for y 0
27

8
We conclude that g ( y )
for y 1
27

otherwise
0

19 8 t
and M(t) = E e yt
 e
27 27

Page 42 of 67

99.

Solution: C
We use the relationships Var aX  b
Var X  Y

a 2 Var X , Cov aX , bY

Var X  Var Y  2 Cov X , Y . First we observe

ab Cov X , Y , and

17, 000 Var X  Y 5000  10, 000  2 Cov X , Y , and so Cov X , Y 1000.
We want to find Var X  100  1.1Y

Var X  1.1Y  100

Var > X  1.1Y @ Var X  Var 1.1 Y  2 Cov X ,1.1Y

Var X  1.1 Var Y  2 1.1 Cov X , Y 5000  12,100  2200 19,300.


2

-------------------------------------------------------------------------------------------------------100.

Solution: B
Note
P(X = 0) = 1/6
P(X = 1) = 1/12 + 1/6 = 3/12
P(X = 2) = 1/12 + 1/3 + 1/6 = 7/12 .
E[X] = (0)(1/6) + (1)(3/12) + (2)(7/12) = 17/12
E[X2] = (0)2(1/6) + (1)2(3/12) + (2)2(7/12) = 31/12
Var[X] = 31/12 (17/12)2 = 0.58 .

-------------------------------------------------------------------------------------------------------101.

Solution: D
Note that due to the independence of X and Y
Var(Z) = Var(3X  Y  5) = Var(3X) + Var(Y) = 32 Var(X) + Var(Y) = 9(1) + 2 = 11 .

-------------------------------------------------------------------------------------------------------102.

Solution: E
Let X and Y denote the times that the two backup generators can operate. Now the
variance of an exponential random variable with mean E is E 2 . Therefore,
Var > X @ Var >Y @ 102 100
Then assuming that X and Y are independent, we see
Var > X+Y @ Var > X @  Var > Y @ 100  100 200

Page 43 of 67

103.

Solution: E
Let X 1 , X 2 , and X 3 denote annual loss due to storm, fire, and theft, respectively. In
Max X 1 , X 2 , X 3 .

addition, let Y
Then

Pr >Y ! 3@ 1  Pr >Y d 3@ 1  Pr > X 1 d 3@ Pr > X 2 d 3@ Pr > X 3 d 3@

1  1  e 3 1  e

1  e
1  e

3
1.5

1  1  e 3 1  e 2

3

5

<

2.4

0.414
* Uses that if X has an exponential distribution with mean P
f

Pr X d x 1  Pr X t x 1 
x

e  t P dt 1  e  t P

f
x

1  e x P

-------------------------------------------------------------------------------------------------------104.

Solution: B
Let us first determine k:
1

kxdxdy

1 2 1
kx : 0 dy
0 2
1

k
dy
0 2
1

k
2

Then
E>X @

1 1

2
2 x dydx
0 0

E >Y @

1
0

2 x 2 dx

1 1

y 2 x dxdy

1
0

ydy

0 0

E > XY @

2x 2 ydxdy

2 2 1 2
y :0
6
6
Cov > X , Y @

2 31 2
x :0
3
3
1 2 1 1
y :0
2
2

2 3 1
x y : 0 dy
0 3
1

2
ydy
0 3

1
3

E > XY @  E > X @ E >Y @

1 2 1

3 3 2

1 1

3 3

(Alternative Solution)
Define g(x) = kx and h(y) = 1 . Then
f(x,y) = g(x)h(x)
In other words, f(x,y) can be written as the product of a function of x alone and a function
of y alone. It follows that X and Y are independent. Therefore, Cov[X, Y] = 0 .

Page 44 of 67

105.

Solution: A
The calculation requires integrating over the indicated region.

E X
E Y

2x

2x

E XY

8 2
x y dy dx
3

8 2
xy dy dx
3

2x

Cov X , Y

4 2 2
0 3 x y
1

8 3
0 9 xy

8 2 2
x y dy dx
3

2x

dx
x

2x

dy dx
x

8 2 3
0 9 x y
1

E XY  E X E Y

4 2
2
2
0 3 x 4 x  x dx

4 x dx

8
3
3
0 9 x 8 x  x dx

56 4
0 9 x dx

2x

dx
x

8 2
3
3
0 9 x 8x  x dx
1

56 5
x dx
9

28 56 4
 0.04
27 45 5

-------------------------------------------------------------------------------------------------------106.

Solution: C
The joint pdf of X and Y is f(x,y) = f2(y|x) f1(x)
= (1/x)(1/12), 0 < y < x, 0 < x < 12 .
Therefore,
12 x
12
12
1
y x
x
x 2 12
E[X] = x
=6
dydx
dx dx
12 x
12 0
12
24 0
0 0
0
0
12 x

y
E[Y] =
dydx
12 x
0 0

y2
0 24 x dx
0

12

12

x
0 24 dx

x 2 12
48 0

144
=3
48

12 2
y2
x
x3 12
(12)3
= 24
dx
dx
0 24
0 24 72 0
72
0
Cov(X,Y) = E[XY] E[X]E[Y] = 24  (3)(6) = 24 18 = 6 .
12 x

E[XY] =

y
0 0 12 dydx

12

Page 45 of 67

4 5
x
5

4
5

56 5
x
45
56
54

56
45
28
27

107.

Solution: A
Cov C1 , C2

Cov X  Y , X  1.2Y

Cov X , X  Cov Y , X  Cov X ,1.2Y  Cov Y,1.2Y


Var X  Cov X , Y  1.2Cov X , Y  1.2VarY
Var X  2.2 Cov X , Y  1.2VarY

Var X

E X 2  E X

Var Y

E Y 2  E Y

Var X  Y

27.4  52
51.4  7 2

2.4
2.4

Var X  Var Y  2 Cov X , Y


1
Var X  Y  Var X  Var Y
2
2.4  2.2 1.6  1.2 2.4 8.8

Cov X , Y
Cov C1 , C2

1
8  2.4  2.4 1.6
2

-------------------------------------------------------------------------------------------------------107.

Alternate solution:
We are given the following information:
C1 X  Y
C2

X  1.2Y

E>X @ 5
E X 2

27.4

E Y 2

51.4

E >Y @ 7
Var > X  Y @ 8
Now we want to calculate
Cov C1 , C2 Cov X  Y , X  1.2Y
E X  Y X  1.2Y  E > X  Y @< E > X  1.2Y @

E X 2  2.2 XY  1.2Y 2  E > X @  E >Y @ E > X @  1.2 E >Y @


E X 2  2.2 E > XY @  1.2 E Y 2  5  7 5  1.2 7

27.4  2.2 E > XY @  1.2 51.4  12 13.4


2.2 E > XY @  71.72

Therefore, we need to calculate E > XY @ first. To this end, observe

Page 46 of 67

8 Var > X  Y @

2
2
E X  Y  E > X  Y @

E X 2  2 XY  Y 2  E > X @  E >Y @
E X 2  2 E > XY @  E Y 2  5  7

27.4  2 E > XY @  51.4  144


2 E > XY @  65.2

E > XY @

8  65.2 2 36.6
Finally, Cov C1,C2 2.2 36.6  71.72

8.8

-------------------------------------------------------------------------------------------------------108.

Solution: A
The joint density of T1 and T2 is given by
f t1 , t2 e  t1 e t2 , t1 ! 0 , t2 ! 0
Therefore,
Pr > X d x @ Pr > 2T1  T2 d x @
x

1
x  t2
2
0

e  t1 e t2 dt1dt2

 t1 12 x t2
 t2
e
0 e 0 dt2

1
1
 x  t2
x
 t2
2

e
e
e 2 dt2

1 1
 x  t2


0 e 1 e 2 2 dt2
1
1
1
1
1
 x  t2
 x  x
 x
 t2
x
x
2
2
2
2
2






e
2
e
e
e
2
e
e
1
2
e

 t2

1  e  x  2e  x  2e 2 1  2e 2  e  x , x ! 0
It follows that the density of X is given by
1
1
 x
 x

d
x
2
2


 e x , x ! 0
g x
e
e
e
1
2

dx

Page 47 of 67

109.

Solution: B
Let
u be annual claims,
v be annual premiums,
g(u, v) be the joint density function of U and V,
f(x) be the density function of X, and
F(x) be the distribution function of X.
Then since U and V are independent,
1
1 u  v / 2
g u, v eu e v / 2
e e
, 0/ u / f , 0/ v/ f
2
2
and
u

F x Pr > X d x @ Pr d x Pr >U d Vx @
v

f vx
f vx 1
u  v / 2
0 0 g u, v dudv 0 0 2 e e dudv
f
f
1  u  v / 2 vx
1  vx  v / 2 1  v / 2
e
e
dv

:
0
0 2
0  2 e e  2 e dv

1  v x 1/ 2 1  v / 2
 e
 e
dv
2
2

Finally, f x

 v x 1/ 2
 e v / 2
2 x  1 e
0
2
F ' x
2
2 x  1

1
1
2x  1

-------------------------------------------------------------------------------------------------------110.

Solution: C
Note that the conditional density function
1 f 1 3, y
2

f y x
, 0 y ,

3
f x 1 3
3

1
fx
3

23
0

24 1 3 y dy

It follows that f y x

2 3
0

23

8 y dy

1 9
f 1 3, y
3 16

Consequently, Pr Y  X X

1 3

16
9

4 y2

13
0

9
2
y , 0 y
2
3

9
y dy
2

Page 48 of 67

9 2
y
4

13

1
4

111.

Solution: E
Pr 1  Y  3 X

3
1

f 2, y
dy
f x 2

2
 4 1 2 1
y
4 2  1

f 2, y
f x 2

1 3
1 2 y dy

1 3
y
2
f

1
 y 2
4
1

Finally, Pr 1  Y  3 X

1
4

3
1

1 3
y dy
2
1
4

 y 2

3
1

1

1
9

8
9

-------------------------------------------------------------------------------------------------------112.

Solution: D
We are given that the joint pdf of X and Y is f(x,y) = 2(x+y), 0 < y < x < 1 .
x

Now fx(x) = (2 x  2 y )dy


0

2 xy  y 2 = 2x2 + x2 = 3x2, 0 < x < 1


0

2( x  y )
3x 2

21 y
 , 0 < y < x
3 x x2
2 1
y 2

f(y|x = 0.10) =
>10  100 y @ , 0 < y < 0.10
3 0.1 0.01 3
0.05
2
100 2 0.05
20
y
P[Y < 0.05|X = 0.10] = >10  100 y @ dy y 
3
3
3
0
0

so f(y|x) =

f ( x, y )
f x ( x)

1 1

3 12

5
= 0.4167 .
12

-------------------------------------------------------------------------------------------------------113.

Solution: E
Let
W = event that wife survives at least 10 years
H = event that husband survives at least 10 years
B = benefit paid
P = profit from selling policies
Then Pr > H @ P > H W @  Pr H W c 0.96  0.01 0.97
and
Pr >W : H @
Pr W c : H

Pr >W H @
Pr > H @

0.96
0.97

Pr H W c
Pr > H @

0.9897

0.01
0.97

0.0103

Page 49 of 67

It follows that
E > P @ E >1000  B @ 1000  E > B @ 1000  0 Pr >W : H @  10, 000 Pr W c : H
1000  10, 000 0.0103 1000  103 897

-------------------------------------------------------------------------------------------------------114.

Solution: C
Note that
P(Y = 0~X = 1) =

P( X 1, Y 0)
P( X 1)

P( X

P( X 1, Y 0)
1, Y 0)  P ( X 1, Y

1)

0.05
0.05  0.125

= 0.286
P(Y = 1~X=1) = 1 P(Y = 0 ~ X = 1) = 1 0.286 = 0.714
Therefore, E(Y~X = 1) = (0) P(Y = 0~X = 1) + (1) P(Y = 1~X = 1) = (1)(0.714) = 0.714
E(Y2~X = 1) = (0)2 P(Y = 0~X = 1) + (1)2 P(Y = 1~X = 1) = 0.714
Var(Y~X = 1) = E(Y2~X = 1) [E(Y~X = 1)]2 = 0.714 (0.714)2 = 0.20
-------------------------------------------------------------------------------------------------------115.

Solution: A
Let f1(x) denote the marginal density function of X. Then
f1 x

x 1

Consequently,
f y: x

2 xy : xx 1 2 x x  1  x

2 xdy

f x, y
f1 x

2x

0/ x/1

1 if: x / y / x  1

0 otherwise

1 2 x 1 1
1
1
1 1
2
y :x
x  1  x 2 x 2  x   x 2
2
2
2
2
2 2
x 1
1 3 x 1 1
1
3
E Y 2 : X y 2 dy
y :x
x  1  x3
x
3
3
3
1 3
1 1
1
x  x 2  x   x3 x 2  x 
3
3 3
3
E >Y : X @

Var >Y : X @

x 1

ydy

E Y : X  ^ E >Y : X @`

1
1
x2  x   x2  x 
3
4

1
12

Page 50 of 67

1
1
x  x   x 
3
2
2

x

1
2

116.

Solution: D
Denote the number of tornadoes in counties P and Q by NP and NQ, respectively. Then
E[NQ|NP = 0]
= [(0)(0.12) + (1)(0.06) + (2)(0.05) + 3(0.02)] / [0.12 + 0.06 + 0.05 + 0.02] = 0.88
E[NQ2|NP = 0]
= [(0)2(0.12) + (1)2(0.06) + (2)2(0.05) + (3)2(0.02)] / [0.12 + 0.06 + 0.05 + 0.02]
= 1.76 and Var[NQ|NP = 0] = E[NQ2|NP = 0] {E[NQ|NP = 0]}2 = 1.76 (0.88)2
= 0.9856 .

-------------------------------------------------------------------------------------------------------117.

Solution: C
The domain of X and Y is pictured below. The shaded region is the portion of the domain
over which X<0.2 .

Now observe
Pr > X / 0.2@

0.2

1 x

6 1  x  y dydx

0.2

1 x

1 2

y  xy  2 y dx
0

0.2
0.2
1
1
2
2
2
6 1  x  x 1  x  1  x dx 6 1  x  1  x dx
0
0
2
2

0.2 1
2
3
3
6
 0.8  1 0.488
1  x dx  1  x : 0.2
0
0 2

-------------------------------------------------------------------------------------------------------118.

Solution: E
The shaded portion of the graph below shows the region over which f x, y is nonzero:

We can infer from the graph that the marginal density function of Y is given by
g y

y
y

15 y dx 15 xy

15 y
y

yy

Page 51 of 67

15 y 3 2 1  y1 2 , 0  y  1

or more precisely, g y

15 y 3 2 1  y 1 2 , 0  y  1

otherwise
0

-------------------------------------------------------------------------------------------------------119.

Solution: D
The diagram below illustrates the domain of the joint density f x, y of X and Y .

We are told that the marginal density function of X is f x x 1 , 0  x  1


while f y x y x 1 , x  y  x  1

It follows that f x, y

fx x f y x y x

if 0  x  1 , x  y  x  1
otherwise

Therefore,
Pr >Y ! 0.5@ 1  Pr >Y d 0.5@ 1 

1
x

dydx

1 1
1 1 7
1

1
 x dx 1  x  x 2 0 2 1  

0
0
2
4 8 8
2

2
[Note since the density is constant over the shaded parallelogram in the figure the
solution is also obtained as the ratio of the area of the portion of the parallelogram above
y 0.5 to the entire shaded area.]
1

2
x

dx 1 

Page 52 of 67

120.

Solution: A
We are given that X denotes loss. In addition, denote the time required to process a claim
by T.
3 2 1 3
x , x  t  2 x, 0 d x d 2
x
Then the joint pdf of X and T is f ( x, t ) 8
x 8
0,
otherwise.
Now we can find P[T t 3] =
4
4
2
3
3 2
12 3 2
xdxdt
x
dt
3 t/ 2 8
3 16 t / 2 3 16  64 t dt
= 11/64 = 0.17 .
t
t = 2x
4 2

4
3
2
1

4
12 1 3

t
16 64 3

12
36 27
1 
4
16 64

t=x

1 2

-------------------------------------------------------------------------------------------------------121.

Solution: C
The marginal density of X is given by
fx x

1
2
0 64 10  xy dy
1

1
xy 3

10
y

64
3 0

1
x
10 
64
3
10

Then E ( X )
=

10

x f x ( x)dx

10

1 2 x3
1
x2
5x 
10 x  dx =
64
9 2
64
3

1
1000
8
500 
 20  = 5.778

64
9
9

Page 53 of 67

122.

Solution: D
y

The marginal distribution of Y is given by f2(y) = 6 e e


x

2y

dx = 6 e

2y

Therefore, E(Y) =

0
f

f2(y) dy = (6 ye

2 y

 6 ye

3 y

x

dx

= 6 e2y ey + 6e2y = 6 e2y 6 e3y, 0 < y < f


f

) dy = 6

ye

2 y

dy 6

e3y dy =

6
6
2 ye2y dy  3 y e3y dy

20
30
f

But 2 y e2y dy and 3 y e3y dy are equivalent to the means of exponential random
f

variables with parameters 1/2 and 1/3, respectively. In other words, 2 y e2y dy = 1/2
0

and 3 y e3y dy = 1/3 . We conclude that E(Y) = (6/2) (1/2) (6/3) (1/3) = 3/2 2/3 =
0

9/6  4/6 = 5/6 = 0.83 .


-------------------------------------------------------------------------------------------------------123.

Solution: C
Observe
Pr > 4  S  8@ Pr 4  S  8 N 1 Pr > N 1@  Pr 4  S  8 N ! 1 Pr > N ! 1@
1 4 5 8 5 1 1 2 1
e e
 e e
<
3
6
0.122
*Uses that if X has an exponential distribution with mean P

Pr a d X d b

Pr X t a  Pr X t b

Pe
a

Page 54 of 67

t P

dt 
b

t P

dt

e

124.

Solution: A
First note that f( X ,Y ) ( x, y) e x (2
2e 2 y ) f X ( x) fY ( y) and that the support
of f( X ,Y ) is a cross product. Therefore X and Y are independent. Thus
Var((Y | X 3 and Y
Var((Y | Y 3))
Var((Y 3))
Var((Y ) Var(3)
Var(Y )
E(Y ) 2
(0.5)
( )2
0.25

3))

Independence of X and Y
Memory-less property of exponential
Independence of Y and 3
Var(3) = 0
Exponential variance

--------------------------------------------------------------------------------------------------------------------125.

Solution: E
The support of (X,Y) is 0 < y < x < 1.
f X ,Y ( x, y )

f ( y | x) f X ( x)

2 on that support. It is clear geometrically

(a flat joint density over the triangular region 0 < y < x < 1) that when Y = y
we have X ~ U(y, 1) so that f ( x | y )

for y

x 1.

f X ,Y ( x, y)

2
2 2y

1 y

By computation:

f Y ( y)

1
y

2dx

2 2y

f ( x | y)

f Y ( y)

Page 55 of 67

1
1 y

for y

x 1

126.

Solution: C
Using the notation of the problem, we know that p0  p1

2
and
5

p0  p1  p2  p3  p4  p5 1 .
Let pn  pn 1 c for all n d 4 . Then pn p0  nc for 1 d n d 5 .
Thus p 0  p 0  c  p 0  2c    p 0  5c 6 p 0  15c 1.
Also p0  p1

p0  p0  c

6
5
6 p0  15c 1 . So c
1
12c
5

2 p0  c

2
. Solving simultaneously
5

6 p0  15c 1

2
2 p0  c 5

6 p0  3c

We want p4  p5

1
and 2 p0
60

p0  4c  p0  5c

2 1

5 60
17 15

120 120

25
. Thus p0
60
32
120

25
.
120

0.267 .

-----------------------------------------------------------------------------------------------------------

127.

Solution: D
Because the number of payouts (including payouts of zero when the loss is below the
deductible) is large, we can apply the central limit theorem and assume the total payout S
is normal. For one loss there is no payout with probability 0.25 and otherwise the payout
is U(0, 15000). So,
E[ X ]

0.25 * 0  0.75 * 7500

5625 ,

15000 2
) 56,250,000 , so the variance of one claim is
12
24,609,375 .

E[ X 2 ] 0.25 * 0  0.75 * (7500 2 


Var ( X )

E[ X 2 ]  E[ X ]2

Applying the CLT,

S  (200)(5625)
P  1.781741613 
 1.069044968
(200)(24,609,375)

which interpolates to 0.8575-(1-0.9626)=0.8201 from the provided table.


P[1,000,000  S  1,200,000]

Page 56 of 67

128.

Key: B
Let H be the percentage of clients with homeowners insurance and R be the percentage of
clients with renters insurance.
Because 36% of clients do not have auto insurance and none have both homeowners and
renters insurance, we calculate that 8% (36% 17% 11%) must have renters insurance,
but not auto insurance.
(H 11)% have both homeowners and auto insurance, (R 8)% have both renters and
auto insurance, and none have both homeowners and renters insurance, so (H + R 19)%
must equal 35%. Because H = 2R, R must be 18%, which implies that 10% have both
renters and auto insurance.

129.

Key: B
The reimbursement is positive if health care costs are greater than 20, and because of the
memoryless property of the exponential distribution, the conditional distribution of health
care costs greater than 20 is the same as the unconditional distribution of health care
costs.
We observe that a reimbursement of 115 corresponds to health care costs of 150 (100% x
(120 20) + 50% x (150 120)), which is 130 greater than the deductible of 20.
Therefore, G (115)

130.

F (130) 1  e

130
100

0.727 .

Key: C

>

E 100 0.5

>

100 E 0.5

>

100 E e ln 0.5 X

Page 57 of 67

100 M X ln 0.5 100

1
1  2 ln 0.5

41.9

131. Solution: E

p n1 , n2
,
p1 n1
where p1 n1 is the marginal probability function of N 1 . To find the latter, sum the joint
probability function over all possible values of N 2 obtaining
n1 : p 2|1 n 2 | n1

First, find the conditional probability function of N 2 given N 1

p1 n1

p n1 , n2

n2 1
f

since

e 1  e
 n1

 n1 n2 1

n1 1 f

31

44

e n1 1  e n1

n2 1

n2 1

31

44

n1 1

1 as the sum of the probabilities of a geometric random variable. The

n2 1

conditional probability function is

p n1 , n 2
p1 n1

p 2|1 n 2 | n1

e  n1 1  e  n1

n2 1

which is the probability function of a geometric random variable with parameter p


mean of this distribution is 1 / p 1 / e  n1 e n1 , and becomes e 2 when n1 2 .
132.

e  n1 . The

Solution: C

The number of defective modems is 20% x 30 + 8% x 50 = 10.


10 70

2 3
The probability that exactly two of a random sample of five are defective is
80

5
133. Solution: B
Pr(man dies before age 50) = Pr(T < 50 | T > 40)
=

Pr(40  T  50)
Pr(T ! 40)
e

11.140
1000

e

11.150
1000

11.140
1000

F (50)  F (40)
1  F (40)
1 e

(1.140 1.150 )
1000

= 0.0696
Expected Benefit = 5000 Pr(man dies before age 50) = (5000) (0.0696) = 347.96

Page 58 of 67

0.102 .

134.

Solutions: C
Letting t denote the relative frequency with which twin-sized mattresses are sold, we
have that the relative frequency with which king-sized mattresses are sold is 3t and the
relative frequency with which queen-sized mattresses are sold is (3t+t)/4, or t. Thus, t =
0.2 since t + 3t + t = 1. The probability we seek is 3t + t = 0.80.

Page 59 of 67

135.

Key: E
Var (N) = E [ Var ( N |  )] + Var [ E ( N |  )] = E () + Var () = 1.50 + 0.75 = 2.25

136.

Key: D

1
. Y = 2 implies the first roll is not a 6 and the
6
second roll is a 6. This means a 5 is obtained for the first time on the first roll (probability = 20%)
or a 5 is obtained for the first time on the third or later roll (probability = 80%).

X follows a geometric distribution with p

E >X | X t 3@

137.

1
2
p

62

8 , so E >X Y

2@ 0.2 1  0.8 8 6.6

Key: E

Because X and Y are independent and identically distributed, the moment generating function of X
+ Y equals K2(t), where K(t) is the moment generating function common to X and Y. Thus, K(t) =
-t
t
0.30e + 0.40 + 0.30e . This is the moment generating function of a discrete random variable that
assumes the values -1, 0, and 1 with respective probabilities 0.30, 0.40, and 0.30. The value we
seek is thus 0.70.

Page 60 of 67

138.

Key: D
Suppose the component represented by the random variable X fails last. This is
represented by the triangle with vertices at (0, 0), (10, 0) and (5, 5). Because the density
is uniform over this region, the mean value of X and thus the expected operational time of
the machine is 5. By symmetry, if the component represented by the random variable Y
fails last, the expected operational time of the machine is also 5. Thus, the unconditional
expected operational time of the machine must be 5 as well.

139.

Key: B
The unconditional probabilities for the number of people in the car who are hospitalized
are 0.49, 0.42 and 0.09 for 0, 1 and 2, respectively. If the number of people hospitalized
is 0 or 1, then the total loss will be less than 1. However, if two people are hospitalized,
the probability that the total loss will be less than 1 is 0.5. Thus, the expected number of
people in the car who are hospitalized, given that the total loss due to hospitalizations
from the accident is less than 1 is

0.49
0.42
0.09 0.5
0 
1 
2
0.49  0.42  0.09 0.5
0.49  0.42  0.09 0.5
0.49  0.42  0.09 0.5
140.

0.534

Key: B

Let X equal the number of hurricanes it takes for two losses to occur. Then X is negative
binomial with success probability p = 0.4 and r = 2 successes needed.
P[ X

n  1 r
nr
n]
p (1  p)
r 1

n  1
2
n2

(0.4) (1  0.4)
2  1

(n  1)(0.4) 2 (0.6) n  2 , for n  2.

We need to maximize P[X = n]. Note that the ratio


P[ X n  1]
P[ X n]

n(0.4) 2 (0.6) n 1
(n  1)(0.4) 2 (0.6) n  2

n
(0.6) .
n 1

This ratio of consecutive probabilities is greater than 1 when n = 2 and less than 1
when n  3. Thus, P[X = n] is maximized at n = 3; the mode is 3.

Page 61 of 67

141. Key: C
There are 10 (5 choose 3) ways to select the three columns in which the three items will
appear. The row of the rightmost selected item can be chosen in any of six ways, the row
of the leftmost selected item can then be chosen in any of five ways, and the row of the
middle selected item can then be chosen in any of four ways. The answer is thus
(10)(6)(5)(4) = 1200. Alternatively, there are 30 ways to select the first item. Because
there are 10 squares in the row or column of the first selected item, there are 30  10 = 20
ways to select the second item. Because there are 18 squares in the rows or columns of
the first and second selected items, there are 30  18 = 12 ways to select the third item.
The number of permutations of three qualifying items is (30)(20)(12). The number of
combinations is thus (30)(20)(12)/3! = 1200.
142. Key: B
The expected bonus for a high-risk driver is 0.8 12 (months) 5.00 48 .
The expected bonus for a low-risk driver is 0.9 12 (months) 5.00 54 .
The expected bonus payment from the insurer is 600 48  400 54 50,400 .

Page 62 of 67

143.

Key: E
P(Pr Li) = P(Pr)  P(Li Pr') = 0.10 + 0.01. Subtract from 1 to get the answer.

144.

Key: E
The total time is less than 60 minutes, so if x minutes are spent in the waiting room, less
than 60  x minutes are spent in the meeting itself.

145.

Key: C

f (y | x

0.75)

f (0.75, y )
1

f (0.75, y)dy

f (0.75, y )
.
1.125

Thus,
f (y | x

0.75)

4 / 3 for 0  y  0.5
,

2 / 3 for 0.5  y  1

which leads to Var (Y | X = 0.75) = 11/144 = 0.076.

146.

Key: B
C = the set of TV watchers who watched CBS over the last year
N = the set of TV watchers who watched NBC over the last year
A = the set of TV watchers who watched ABC over the last year
H = the set of TV watchers who watched HGTV over the last year
The number of TV watchers in the set C N A is 34  15  10  7  6  5  4 45 .
Because C N A and H are mutually exclusive, the number of TV watchers in the set
C N A H is 45  18 63 .
The number of TV watchers in the complement of C N A H is thus 100  63 37 .

Page 63 of 67

147.

Key: A
Let X denote the amount of a claim before application of the deductible. Let Y
denote the amount of a claim payment after application of the deductible. Let
O be the mean of X, which because X is exponential, implies that O 2 is the

variance of X and E X

2O2 .

By the memoryless property of the exponential distribution, the conditional


distribution of the portion of a claim above the deductible given that the claim
exceeds the deductible is an exponential distribution with mean O . Given that
E Y 0.9O , this implies that the probability of a claim exceeding the

148.

0.9 2O2 1.8O2 . Then,

deductible is 0.9 and thus E Y

Var Y 1.8O2  0.9O 2

0.99O2 .

Key: C
Let N denote the number of hurricanes, which is Poisson distributed with mean
and variance 4.
Let X i denote the loss due to the ith hurricane, which is exponentially
distributed with mean 1,000 and therefore variance (1,000)2 = 1,000,000.
Let X denote the total loss due to the N hurricanes.
This problem can be solved using the conditional variance formula. Note that
independence is used to write the variance of a sum as the sum of the variances.
Var X Var E X | N  E Var X | N

Var E X 1  ...  X N  E Var X 1  ...  X N


Var NE X 1  E NVar X 1
Var 1, 000 N  E 1, 000, 000 N

1, 0002 Var N  1, 000, 000E N


1, 000, 000(4)  1, 000, 000(4) 8, 000, 000

Page 64 of 67

149.

Key: B
Let N denote the number of accidents, which is binomial with parameters
1
3 and thus has mean 3
4

3
1 3
and variance 3
4
4 4

1
and
4

9
.
16

Let X i denote the unreimbursed loss due to the ith accident, which is 0.3 times
an exponentially distributed random variable with mean 0.8 and therefore
variance (0.8)2 = 0.64. Thus, X i has mean 0.8(0.3) = 0.24 and variance
0.64(0.3) 2

0.0576 .

Let X denote the total unreimbursed loss due to the N accidents.


This problem can be solved using the conditional variance formula. Note that
independence is used to write the variance of a sum as the sum of the variances.

Var X Var E X | N  E Var X | N

Var E X 1  ...  X N  E Var X 1  ...  X N


Var NE X 1  E NVar X 1
Var 0.24 N  E 0.0576 N

0.24

Var N  0.0576E N

9
3
0.0576  0.0576 0.0756.
16
4

Page 65 of 67

150. Key: B
The 95th percentile is in the range when an accident occurs. It is the 75th percentile of the
payout, given that an accident occurs, because (0.95 0.80)/(1 0.80) = 0.75. Letting x
x

be the 75th percentile of the given exponential distribution, F ( x) 1 e 3000 0.75 , so


. Subtracting the deductible of 500 gives 3659 as the (unconditional) 95th
x 4159
4
percentile of the insurance company payout.
151. Key: C
The ratio of the probability that one of the damaged pieces is insured to the probability
r 27 r
1
3
27
4
4r
that none of the damaged pieces are insured is
, where r is the total
r 27 r
24 r
0
4
27
4
number of pieces insured. Setting this ratio equal to 2 and solving yields r = 8.
The probability that two of the damaged pieces are insured is
r

27 r

19

27

27

(8)(
(7)(19)(18)(4)(3)(2)(1)
(8)(7)(19)(18)(4)(3)(2)(1)
(27)((26)(25)(24)(2)(1)(2)(1)
(27)(26)(25)(24)(2)(1)(2)(1)

266
975

0.27 .

152. Key: A
The probability that Rahul examines exactly n policies is 0.1 0.9n 1 . The probability that
Toby examines more than n policies is 0.8n . The required probability is thus
1
0.72
0
72
0.1 0.9n 1 0.8
0.2857 .
0 8n
0.72n
9n 1
9 1 0.72
0 72
n 1
An alternative solution begins by imagining Rahul and Toby examine policies
simultaneously until at least one of the finds a claim. At each examination there are four
possible outcomes:
1. Both find a claim. The probability is 0.02.
2. Rahul finds a claim and Toby does not. The probability is 0.08.
3. Toby finds a claim and Rahul does not. The probability is 0.18
4. Neither finds a claim. The probability is 0.72.
Conditioning on the examination at which the process ends, the probability that it ends
with Rahul being the first to find a claim (and hence needing to examine fewer policies)
is 0.08/(0.02 + 0.08 + 0.18) = 8/28 = 0.2857.
Page 66 of 67

153. KEY: E

Solution:

1. ForaPoissonrandomvariableN,E(N)=Var(N)=andE(N2)=+2.
2. Wearegivenx=y8andx+2x=0.6*(y+2y).
3. Combiningequations,wegetaquadraticiny:y8+(y8)2=0.6*(y+2y).
4. Simplifying,weget:0.42y15.6y+56=0.
5. Tworoots:y=35and4.Thesecondrootisrejectedsincethiswouldmakex
negative.
6. Therefore,thevarianceis35.




Page 67 of 67

You might also like